You are on page 1of 16

GV: Kiều Ngân

Trang 46 - 50
Bài 46 - 55

Bài 46. Dãy được cho bởi điều kiện và . Đặt

. Hãy chứng minh rằng: .

Nhận xét:
Bài toán này nếu không lượng giác hóa thì sẽ đi vào thế bế tắc. Thật vậy, vì phương pháp sai
phân không thể giải quyết bài toán có nhiều căn như vậy.
Bây giờ ta chú ý đại lượng , điều này cho ta một cảm giác gần giống công thức

hay ! Vậy ta tiến hành giải.


Lời giải

Ta có: .

Ta sẽ chứng minh rằng: .

Giả sử rằng:

Mặt khác: ;

Nên:

Do là dãy tăng nên .

đpcm.

Bài 47. Cho dãy và như sau: và

Chứng minh rằng: .


Nhận xét:
Với dãy ta thấy có biểu thức , ta nghĩ ngay đến lượng giác hóa bằng sin, cos.

Còn dãy ? Câu trả lời nằm ở biểu thức và , cho ta suy nghĩ nên sử dụng hàm
tang và cotang.
Lời giải
Ta có: , .

Vậy: .

Tương tự: .

Bằng cách xét: , ; .

Ta suy ra: ; .

Khi đó: đpcm.

Bài 48. Cho cấp số cộng gồm số hạng với số hạng đầu và công sai là .

Tính giá trị: ở đó tổng chứa tất cả các số hạng ứng với tất cả
các cách khác nhau có thể được để lấy dấu cộng hay trừ trước các số , , …, .
Nhận xét:
Ta thấy rằng số là cố ý cho trùng với năm thi. Điều đó chứng tỏ có thể tổng quát hóa bài
toán. Đôi khi việc này khiến bài toán dễ dàng hơn và không còn che giấu như bài toán nguyên
thủy.
Lời giải
Ta sẽ chứng minh từ bài toán tổng quát hơn. Bài toán thực chất là:

(kí hiệu dãy) .

Ta chứng minh bằng quy nạp:


Với :

Với :

Giả sử bài toán đúng với , khi đó:


Trở lại bài toán ta có:

Do là cấp số cộng nên:

Bài 49. Cho dãy số , , ... như sau: , , . Dãy số , , ... được theo quy

luật: . Hãy tìm .

Lời giải
Trước hết nhận xét rằng dãy , , ... chính là các số hạng lẻ của dãy Fibonaci: , , , , ,
…. Gọi dãy đó là , , , …
Ta có: và .
a. Trước hết ta chứng minh rằng:

Thật vậy theo công thức cộng cung ta có:

. Chú ý là:

.
Nếu đặt thì .
Từ đó: .
Suy ra:

Trong lần lượt thay , , , … rồi cộng lại sẽ được .

b. Từ suy ra:

Do: nên .

Từ đó suy ra .

Vậy: .

Bài 50. Cho dãy số được xác định bởi , .Chứng minh rằng dãy

có giới hạn hữu hạn khi . Tìm .


Lời giải
Với ta có

Vì nên

Bài 51. Cho dãy số xác định bởi

a) Chứng minh rằng: , .

b) Chứng minh rằng: , . Tính .

Lời giải

a) Bằng quy nạp dễ dàng chứng minh được , . Ta CM , . Với thì

đúng. Giả sử , , ta chứng minh . Thật vậy, ta có

và . Do đó .

Vậy , .

b) Từ câu a) suy ra ,

Do đó ta có , .

Mà , nên theo nguyên lí kẹp thì .

Bài 52. Cho dãy số xác định bởi

a) CMR: và , .

b) Tính .
Lời giải
Nhận xét: Việc xác định CTTQ của dãy rất khó khăn, nhưng từ hệ thức truy hồi ta thấy có

thể đánh giá tỉ số dễ dàng.

a) Dễ dàng chứng minh bằng quy nạp được , .

Từ hệ thức truy hồi ta có , .


b) Từ câu a) ta có .

Mà . Nên theo nguyên lí kẹp ta có .

Bài 53. Cho dãy số xác định bởi . Tính .

Lời giải
Nhận xét: Việc xác định CTTQ của dãy thật không đơn giản, nhưng ta thấy rằng ,
với mọi (kiểm tra bằng quy nạp). Hơn nữa theo bất đẳng thức Cosi, ta có
.

Dấu “=” không xảy ra vì , , do đó , .

, (*)
Áp dụng (*) liên tiếp nhiều lần ta có
,

Hay , .

Mà nên theo nguyên lí kẹp ta có .

Bài 54. Cho dãy số xác định bởi . (với ).

a) Chứng minh rằng: , .

b) Tính .
Lời giải
Nhận xét rằng , với mọi (kiểm tra bằng chứng minh quy nap). Từ đó suy ra
và .

Suy ra , nên Dãy là dãy giảm.

Do đó ,

Nên ,

, .
Hay ,

Vì .

Do đó theo nguyên lí kẹp ta được .


xn 2
Bài 55.Cho dãy số xn  xác định bởi x1  1; 2  và xn 1  1  xn – . Chứng minh rằng xn  có giới
2
hạn hữu hạn khi n dần đến vô cùng và tìm giới hạn đó.
2
a
Lời giải. Giả sử xn có giới hạn là a thì a  1  a – từ đó suy ra a  2 Ta sẽ dùng định
2
nghĩa để chứng minh lim xn  2 .
Ta có
xn 2 2  xn  1
xn 1  2  1  xn –  2  xn  2 .
2 2
3
Tiếp theo ta có thể chứng minh bằng quy nạp rằng 1  xn < với mọi n  2,3,  .Từ đó, do
2
1
2  2 nên suy ra lim xn  2 .
2
Bài 56.Xét dãy số với số hạng tổng quát như sau:
1 1 1
1. un  1    ...   2 n, n  
2 3 n
1 1 1
2. un  1    ...  , n  
1! 2! n!
 1  1   1  1 
3. un  1  2    1  2    1  2   ...   1  2  , n  
 1   2   3   n 
Chứng minh rằng các dãy số trên nếu có giới hạn hữu hạn.
Bài giải
Ta có
 1 1 1  1 1 1 
un 1  un  1    ...   2 n  1  1    ...   2 n 
 2 3 n 1  2 3 n 
 
 1  n  n 1
  2 n 1  2 n    0
 n 1
 
  n 1 n 1  n 
 
 un1  unn    un là dãy giảm.

1 1 1
mặt khác k  1  k    (1)
k 1 k 1  k 2 k
ltrong (1) lần lượt thay k  1, 2,3...n ta có:
1
 2 2 2 1
1
1
 2 32 2
2
...
1
 2 n 1  2 n
n
Cộng tổng vế các bất đẳng thức trên ta có:

1 1 1
un  1    ...   2 n  2
2 3 n
Như vậy un  bị chặn bởi 2 , theo nguyên lý giới hạn dãy số un  tồn tại giới hạn.

1 1 1
un  1    ...  , n  
1! 2! n!
Rõ ràng un 1  un nên un  là dãy tăng.
Ta chứng minh rằng un  2 .
1 1 3
thật vậy ta có: u1  1 ; u2    .
1! 2! 2
chú ý rằng 3!  2.3  2 2

4!  2.3.4  23
.
n !  2.3.4.5...n  2n 1
n
1
1  
1 1 1 1 1
Vậy un  1    ...   1   2  ...  n 1 
1  2   2(*) .
1! 2! n! 2 2 2 1
1
2
un  là dãy tăng và bị chặn trên nên un  có giới hạn hữu hạn.

 1  1   1  1 
un  1  2   1  2   1  2   ...   1  2  , n   .
 1   2   3   n 
Dễ dàng chứng minh được nhận xét sau đây:
Nếu r là số hữu tỷ dương thì r  1  3r .
xét dãy
 1  1  1  1
un  1   . 1    1    ...  1   , n   .
 1!   2!   3!   n! 
ta thấy un là dãy tăng và
1
 1  1!
1    3
 1! 
1
 1
.1    3 2!

 2! 
 1  3!1
1    3
 3! 
...
1
 1
 1    3 n!

 n! 
Ta thấy un  là dãy tăng và un  3 .
2

Vậy un  là dãy tăng và bị chặn bởi 9 nên un  có giới hạn hữu hạn.
Bài 57.Cho dãy số: un  xác định như sau:
 1
 u1 
3
 2
n  *
u  u n  1
 n 1 2
Tìm lim un
Bài giải

Ta nhận thấy
n  2 thì –1  un  0 .từ đó suy ra nếu un  có giới hạn là a thì 1  a  0 .

a2  a  1  3(lL)
và a thõa a   1  a 2  2a  1  0  
2  a  1  3
Xét

  
2
u n   1 3
   
2 2
u n
un 1  1  3  1 1 3    1   1
2  
 2   1 
 

 
2
1 3
   
2
u n un
   1  3  un  1  3
2 2 2

Do –1  un  0 nên

Do un  0 và 1  3  0 nên

 
un  1  3  un  1  3  un  3  1  3 ;  un 1  1  3  3 u2  1  3
2
   
n
lặp lại bất đẳng thức (*) đến lần ta có:
n n
 3  3

 un 1  1  3   
 u2  1  3    
 n  1;2...
 2   2 

n
 3

như thế ta có: 0  un 1  1  3    0
 2 
theo nguyên lý kẹp ta có lim un  1  3 .

1
Bài 58.Cho dãy số (an ) được xác định bởi an   1 với mọi n  1, 2,3,... Chứng minh rằng tồn tại số
n2
a2 a3 a
nguyên dương n0 sao cho với mọi n  n0 thì   ...  n 1  n  2009 .
a1 a2 an
an 1
Lời giải: Đặt bn  1  với mọi n  1, 2,3,... Khi đó ta có:
an
1 1 1  
1  1 1 1 1 
n 2
(n  1)2  1 1  n2 2n  1 1 2n 
bn   n2  2  2 
   
1
1 1  n (n  1)  1  1  1
1
(n  1)2 2 1  2(n 
2 2 1 

n2 n2 ( n  1)2  n 
1
Lại có (2n  1)(n  2)  2( n  1) 2  n  0 nên bn  với mọi n  1, 2,3,...
n2
Đặt S n  b1  b2  ...  bn với mọi n  1, 2,3,...
Nếu n  2k  2  1 (k  * ) , áp dụng chứng minh trên ta được:
1 1 1 1  1 1   1 1   1 1 
Sn    ...  k  k   2  2  ...  3   ...   k 1  ...  k 
3 4 2 1 2  2 1 2   2 1 2   2 1 2 
1 1 1 k 1
 2. 2  22. 3  ...  2k 1. k  .
2 2 2 2
ak 1
Chọn n0  2  2 , khi đó với mọi n  n0 , để ý rằng 1   0 k  1, 2,... nên ta có:
4019
ak
4019  1 a a a
S n  S n0   2009  2  3  ...  n 1  n  2009 .
2 a1 a2 an
Vậy ta có điều phải chứng minh.
a0  x , a1  y

Bài 59.Cho dãy số (an ) được xác định bởi:  an an 1  1
 an 1  n  1, 2,3,...
 an  an 1
( 1 ) Tìm tất cả số thực x và y sao cho tồn tại số nguyên dương n0 mà với mọi n  n0 thì an là
hằng số.
( 2 ) Tìm công thức tổng quát tính an .
Lời giải: ( 1 ) Ta có:
an an 1  1 a2 1
an  an 1  an   n với mọi n  1, 2,3,...
an  an 1 an  an 1
Do đó nếu tồn tại số nguyên dương n sao cho an  a n 1 thì an  1 và an  an 1  0 .
2
Nếu n  1 , ta có | y | 1 và x   y . ( I )
an 1an  2  1 (a  1)(an  2  1)
Nếu n  1 , ta có : an  1   1  n 1 ( II )
an 1  an  2 an 1  an  2
an 1an  2  1 (a  1)(an  2  1)
Và an  1   1  n 1 ( III )
an 1  an  2 an 1  an  2
an21  1 a2 1
Từ ( II ) và ( III ) ta có : an  1  . n2
2
( IV )
an 1  an  2 an 1  an  2
Vậy an 1  1 hoặc an  2  1 , cứ tiếp tục lui dần cho đến khi ta được a0  1 hoặc a1  1 , tức là ta
2 2 2 2

có :
 | x | 1

 | y | 1 ( * ). Thử lại nếu x, y thỏa mãn điều kiện ( * ) thì an  c với mọi n  2,3,... trong
x  y  0

đó c nhận giá trị là 1 hoặc 1 .
 | x | 1

Vậy các số x, y cần tìm chính là  | y | 1 .
x  y  0

an  1 an 1  1 an  2  1 an  1
( 2 ) Từ ( II ) và ( III ), ta có :  . với mọi n  2,3,... Đặt bn  thì ta
an  1 an 1  1 an  2  1 an  1
được :
bn  bn 1.bn  2 . Bằng phép quy nạp đơn giản, ta thu được : bn  b0Fn2 .b1Fn1 với mọi n  2,3,...
Fn1 Fn2
a 1  y 1   x 1  ( x  1) Fn2 .( y  1) Fn1  ( x  1) Fn2 .( y  1) Fn1
Hay ta có n   .   an  .
an  1  y  1   x 1  ( x  1) Fn2 .( y  1) Fn1  ( x  1) Fn2 .( y  1) Fn1
Trong đó Fn là số hạng thứ n của dãy Fibônaci.
2 n  2006
ak 1
Bài 60.Cho dãy số (an ) bị chặn và thỏa mãn: an  
k n

k  1 2n  2007
với mọi n  1, 2,3,...

1
Chứng minh rằng an  với mọi n  1, 2,3,...
n
2 n  2006
1 bk
Lời giải : Đặt bn  an  . Khi đó theo giả thiết ta có bn   với mọi n  1, 2,3,...
n k n k  1

Vì dãy (an ) bị chặn nên tồn tại số thực M sao cho bn  M n .


 3n 
2 n  2006 2 n  2006 2 2 n  2006
 
bk 1 1 1
Với n  100000 , ta có bn  
k n k 1
 M. 
k n k 1
 M .
k n k  1
 M. 
 3n  k  1
k    1
2

n
 2006
1 6
 M.  M. 2  M.
2 3n 7
1
2
m
6  6
n
Tiếp tục làm như vậy khi thay M bởi M , ta sẽ thu được với đủ lớn thì bn    .M với
7 7
mọi m  * .
m 2 n  2006
6 bk
Lại có lim    0 nên n
m 7
b  0 n
với đủ lớn. Mà n b   k  1
với mọi n  1, 2,3,... nên
  k  n

1
bn  0 với mọi n  1, 2,3,... Vậy an  với mọi n  1, 2,3,... Đây cũng chính là điều phải
n
chứng minh.
 x1  x2  0; x3  9
Bài 61. Cho dãy số ( xn )  xn  được xác định bởi  . Tìm
 xn  xn 1  xn  2  3xn  2 n  1
lim xn .
n 

Lời giải
Đặt an  max{xn ; xn 1 ; xn  2 } và bn  min{xn ; xn 1 ; xn  2 } .
Giả sử tồn tại n sao cho an 1  an  an 1  xn 1 , xn  2
 an 1  xn 3  3xn 3  xn  xn 1  xn  2  3xn 3 (vô lý )
Vậy an  an 1 n  1 .
Hoàn toàn tương tự ta có bn  bn 1 n  1 . Vậy ta có dãy các đoạn đan lồng nhau :
[b1 ; a1 ]  [b2 ; a2 ]  ...  [bn ; an ]  ... Đặt d n  an  bn thì theo trên (d n ) là dãy giảm.
 xn 3  xn  an 3  bn  an  bn  d n
Ta có  | xn 3  xn | d n
 xn 3  xn  bn 3  an  bn  an  d n
x x x x x x x x d
| xn  4  xn 3 || n 1 n  2 n 3  n n 1 n  2 || n 3 n | n
3 3 3 3 .
d 2
Tương tự | xn 5  xn  4 | n . Do đó ta có | xn 5  xn 3 || xn 5  xn  4 |  | xn  4  xn 3 | d n
3 3
2
 d n 3  an 3  bn 3  d n .
3
n 3 n 3
2 2 dn  0 .
Từ đó ta dễ dàng có được d    d1d 2 d3 . Vì lim   d1d 2 d3  0 nên lim
3
n n 
3 n  3
   
Vậy các đoạn thắt [bi ; ai ] bi ; ai  có một điểm chung c duy nhất.
bn  xn  an
Ta có  | xn  c | an  bn  d n . Vì lim d n  0 nên lim xn  c .
bn  c  an
n  n 

1 2
Dễ dàng quy nạp từ giả thiết rằng xn  2  9  xn  xn 1 n  1 , chuyển qua giới hạn
3 3
9
ta được lim xn  c 
n  2
Bài 62. Cho dãy số ( xn ) thỏa mãn điều kiện với mọi số nguyên dương m, n thì
1
| xm  n  x n  xm | .
mn
Chứng minh rằng ( xn ) là một cấp số cộng.
Lời giải
Với mọi số nguyên dương k , n , theo giả thiết ta có :
| ( xn 1  xn )  ( xk 1  xk ) || ( xn  k 1  xn  xk 1 )  ( xn  k 1  xk  xn 1 ) || xn  k 1  xn  xk 1 |  | xn  k 1  xk  xn 1 |
1 1 2
   .
n  k 1 n  k 1 n
Cố định k và cho n   , ta có lim( xn 1  xn )  xk 1  xk k  1, 2,3,...
n 

Vậy dãy ( xn ) là một cấp số cộng với công sai là lim( xn 1  xn ) .


n 
Đó cũng chính là điều phải chứng minh.
n
1
Bài 63. Dãy số (un ) được xác định bởi công thức un   2
n  1 . Chứng minh rằng dãy
k 1 ( k !)

(un ) có giới hạn hữu hạn và giới hạn đó là một số vô tỷ.


Lời giải
1
Vì  0 k  1, 2,3,... nên un là dãy đơn điệu tăng.
(k !) 2
1 1 1 1 1
Với mọi số nguyên dương k thì     , do đó ta có :
(k !) 2
k ! k (k  1) k  1 k
n
1 1 n
1 1 n  1 1 1 1 1 7
un    1     1       1     với mọi số
k 1 ( k !)
2
4 k 3 (k !) 2
4 k 3  k  1 k  4 2 n 4
nguyên dương . n
7
Vậy (un ) bị chặn trên bởi và là dãy đơn điệu tăng nên tồn tại giớn hạn hữu hạn
4
lim un  a   .
n 

Xét dãy (vn ) được xác định như sau , với mọi số nguyên dương n thì
1
vn  un  .
(n !) (n  1)
2

(n  1)(n 2  2n  1)
vn 1  vn   0 n  1, 2,3,... . Vậy (vn ) là dãy đơn điệu giảm, dễ
((n  1)!) 2 (n  1)(n  2)
thấy nó bị chặn dưới bởi 0 nên tồn tại giới hạn hữu hạn lim vn  b . n 

1
vn  un   lim vn  lim un  a  b . Giả sử phản chứng rằng a là một số
(n !) (n  1)
2 n  n 

p
hữu tỷ, tức là a  , p, q  Z . Theo các chứng minh trên , ta có với mọi n nguyên
q
dương thì :
n
p 1 p n 1 1
un   vn     
q k 1 ( k !)
2
q k 1 (k !) (n !) (n  1)
2 2

n
(n !) 2 (n  1) p(n !) 2 (n  1) n (n !) 2 (n  1)
   1 ( * )
k 1 (k !) 2 q k 1 (k !)2
p(n !) 2 (n  1) n (n !) 2 (n  1)
Với n đủ lớn thì , là các số nguyên, do đó từ ( * ) cho ta
q k 1 (k !) 2
có một số nguyên nằm giữa hai số nguyên liên tiếp , đó là điều vô lý. Vậy điều giả sử là
sai , tức là a là một số vô tỷ.
Vậy bài toán được chứng minh hoàn toàn.
 x1  x2  1

Bài 64. Cho dãy số ( xn ) được xác định bởi  xn . Tìm lim xn .
x
 n  2  x 2
n 1   n  1, 2,3,... n 

2
Lời giải
1
Ta sẽ chứng minh bằng quy nạp rằng xn  n  5 ( * )
n
Với n  6, 7 , dễ thấy ( * ) đúng.
Giả sử ( * ) đúng đến k  1  n , ta sẽ chứng minh ( * ) đúng đến k  2 .
x x 1 1 1
Thât vậy , ta có | xk  2 || xk 1  k | xk 1  k    k  5 .
2 2

2 2 (k  1) 2
2k k  2
Vậy ( * ) đúng đến k  2 , theo nguyên lý quy nạp thì nó đúng với mọi n  5 .
1
Mà lim  0  lim xn  0 .
n  n n 

Bài 65. Cho dãy số dương (an ) thỏa mãn các điều kiện :
1 ) an 1  an  an
2

2) Tồn tại một số dương M sao cho a1  a2  ...  an  M với mọi n  1, 2,3,...
Tìm lim(nan ) .
n 

Lời giải
Đặt bn  a1  a 2 ...  an , do ak  0 k  1, 2,3,... nên (bn ) là dãy tăng thực sự. Lại theo
giả thiết 2) thì (bn ) bị chặn trên bởi M , vậy tồn tại giới hạn hữu hạn lim bn  a .
n 

Đặt cn  b2 n  bn  an 1  an  2  ...  a2 n , khi đó ta có lim cn  lim b2 n  lim bn  a  a  0 .


n  n  n 

ak 1 a2 n 2 n 1 ak 1 2 n 1
a
Đặt t  min a k , từ điều kiện 1) ta được  1  ak     (1  ak )
1 k  n ak at k t ak k t

Áp dụng bất đẳng thức quen thuộc 1  x  e x x  0 , ta thu được


2 n 1

a2 n  ak
 e k t  ecn  a2 n  ecn .at
at
1 b
Lại có theo cách xác định at thì at  (a1  a2  ...  an )  n , do đó theo chứng minh
n n
trên ta được
cn .ecn
a2 n   0  2n.a2 n  2cn ecn .
n
Lại có lim 2cn ecn  0 ( do lim cn  0 ) do đó lim 2n.a2 n  0 .
n  n  n 

Hoàn toàn tương tự, ta cũng có lim(2 n  1).a2 n 1  0 .


n 

Vậy lim nan  lim 2n.a2 n  lim(2n  1).a2 n 1  0 .


n  n  n 

1 2
Bài 66. Dãy số ( xn ) thỏa mãn điều kiện 1  x1  2 và xn 1  1  xn  xn , n  * .
2
Chứng minh rằng dãy số đã cho hội tụ. Tìm lim
n 
xn

Lời giải
1
Ta sẽ chứng minh bằng quy nạp bất đẳng thức sau: xn  2  , n  3 . Thật vậy ta
2n
kiểm tra được ngay bất đẳng thức đúng với n  3 . Giả sử bất đẳng thức đúng với n  3 ,
1
tức là xn  2  n . Khi đó ta có
2
1 1
xn 1  2  xn  2 2  2  xn  xn  2
2 2

2  xn  2  2 2 
1 1 1 1
 xn  2  n
 n 1 .
2 22 2
1
Do đó bất đẳng thức đúng đến n  1 . Mặt khác do lim n  0 nên từ bất đẳng thức trên
2
và nguyên lý kẹp ta có lim xn  0 .
Bài 67. Cho dãy các hàm số Pn ( x) xác định như sau
x  Pn2 ( x)
P0 ( x)  0, Pn 1 ( x)  Pn ( x)  , n  0; x   .
2
Tìm lim
n 
Pn ( x) .
Lời giải
Trước hết ta chứng minh bằng quy nạp bất đẳng thức sau: 0  Pn ( x)  x , n   .
(1)
x
Thật vậy, với x  [0,1] suy ra x  2 x  0 nên 0   P1 ( x)  x . Như vậy (1) đúng
2
1
với n=1. Giả sử (1) đúng đến $n$. Xét hàm số f (t )  t   x  t  với t  [0,1] . Dễ thấy
2

2
hàm số f (t ) đồng biến trên [0,1] . theo giả thiết quy nạp ta có 0  Pn ( x)  x  1 với
mọi x  [0,1]
(2)
nên Pn 1 ( x)  f ( Pn ( x))  f ( x )  x với mọi x  [0,1] . Mặt khác, từ (2) ta có
x  Pn2 ( x)  0  Pn 1 ( x)  Pn ( x)  0 . Vậy 0  Pn 1 ( x)  x . Do đó (1) đúng đến n  1
nên theo nguyên lý quy nạp ta có (1) đúng với mọi n .
2
Tiếp theo ta chứng minh x  Pn ( x)  với x  [0,1], n   .
n 1
(3)
 x  Pn 1 ( x) 
Thật vậy ta có x  Pn ( x)   x  Pn 1 ( x)  1  
 2 
 x
  x  Pn 1 ( x)  1   (do Pn 1 ( x)  0)
 2 
n n
 x 2n x x
 ...   x  P0 ( x)  1    1  
 2  n 2  2 
n 1
n x  x 
  n 1   n
2 2  2  2  n  2
       .
n n 1 n 1  n 1  n 1
 
 
2
Từ đó ta thu được bất đẳng thức 0  x  Pn ( x)  với mọi x  [0,1]n   .
n 1
2
Do lim  0 nên theo nguyên lý kẹp ta được lim Pn ( x)  x , với mọi x  [0,1] .
n 1

Cho a, b   , (a, b)  1; n  ab  1, ab  2,... . Kí hiệu rn là số cặp số (u , v)  å  å


å
Bài 68.
r 1
sao cho n  au  bv . Chứng minh rằng lim n  .
n  n ab

Lời giải
Xét phương trình au  bv  n (1). Gọi (u0 , v0 ) là một nghiệm nguyên dương của (1).
Giả sử (u , v) là một nghiệm nguyên dương khác (u0 , v0 ) của (1). Ta có
au0  bv0  n, au  bv  n suy ra a (u  u0 )  b(v  v0 )  0 do đó tồn tại k nguyên dương
v0  1
sao cho u  u0  kb, v  v0  ka . Do v là số nguyên dương nên v0  ka  1  k  .
a
(2)
Ta nhận thấy số nghiệm nguyên dương của phương trình (1) bằng số các số k nguyên
 v  1  n u 1
dương cộng với 1. Do đó rn   0   1    0    1 . Từ đó ta thu được bất
 a   ab b a 
đẳng thức sau:
n u0 1 n u0 1
   rn     1.
ab b a ab b a
Từ đó suy ra
1 u0 1 rn 1 u0 1 1
       .
ab nb na n ab nb na n
r 1
Từ đây áp dụng nguyên lý kẹp ta có ngay lim n  .
n  n ab

Bài 69. Tìm giới hạn của dãy số ( xn ) biết x  1  2 1  3 1   1  ( n  1) 1  n


n

Lời giải 1.
x 1
Xét hàm số f ( x)  1  x 1  (1  x)  ta chứng minh  f ( x)  2( x  1) . Từ
2
1 1
đó 2 2n (1  x)  f ( x)  2 2n (1  x) .
1 1
Từ đó, thay x  2 được 3·2 2n  x  3·2 2n . Từ đó, theo nguyên lý kẹp, suy ra
n

lim xn  3 .
n 

Lời giải 2
Với 1  m  n  1 , đặt a  1  m 1  (1  m) 1   1  (n  1) 1  n ta có
m

am2  1  mam 1  am2  (m  1) 2  mam 1  m 2  2m


.
 am2  (m  1) 2  m(am 1  (m  2))
Suy ra
m | am 1  am  2 | m
| am  (m  1) |  | am 1  m  2 | .
| am  (m  1) | m  2
n 1 n 1
Từ đó | a2  3 | | an 1  n | | 1  (n  1) 1  n  n | 0 ( n  )
n 1 n 1
Lời giải 3
Để ý rằng

3  1  2·4  1  2· 16 n&  1  2 1  3 25  1  2 1  3 1  4 36 bằng quy nạp,


dễ dàng chứng minh được

1  2 1  3 1   1  n ( n  2) 2  3
Suy ra xn  3
(1)

Nhận xét. Cho   1 . Khi đó 1   x   · 1  x x  0 .


Áp dụng nhận xét trên với x  n,   n  2 được
1  n (n  2) 2  n  2· n  1 .
Từ đó
1  (n  1) 1  n (n  2) 2  1  n  2·(n  1)· 1  n  4 n  2· 1  (n  1) 1  n .
n
Do đó, bằng quy nạp, thu được 3  (n  2) 2 xn (2)

Từ (1),(2) và nguyên lý kẹp, suy ra lim xn  3 .


n 

Bài 70. Cho dãy số thực {xn} xác định bởi x0  1, x n 1  2  x n  2 1  x n với mọi n  N.
n
Ta xác định dãy {yn} bởi công thức y n   xi 2 , n  N . Tìm công thức tổng quát của
i *

i 1
dãy {yn}.
Lời giải
Ta có
x n 1  2  x n  2 1  x n  ( 1  x n  1) 2
Từ đó tính được
   
2
2  1 , x 2   2  1 ,..., x n  21 / 2  1
2 n 2
x1 
 
Ta viết
x1  1  2  2 2 ,
x 2  1  2  2.21 / 4
x3  1  21 / 4  2.2.1 / 8
...
n 1 n
x n  1  21 / 2  2.21 / 2
Nhân đẳng thức đầu với 2, đẳng thức thứ hai với 22, đẳng thức thứ ba với 23 … đẳng
thức thứ n với 2n rồi cộng vế theo vế, chú ý đến những sự giản ước, ta được.
n n
y n  2  4  ...  2 n  4  2 n 1.21 / 2  2 n 1 (1  21 / 2 )  2 .

You might also like